Question 1
A store owner want to develop a new snack mix by mixing chocolate and trail mix. How many pounds of
chocolate costing $8.20 per pound should be mixed with trail mix costing $3.40 per pound to create a 23
mixture worth $5.49. (round to the nearest pound)

Answers

Answer 1

The number of pounds of chocolate costing $8.20 per pound should be mixed with trail mix is; 18 pounds

How to solve Algebra Word Problems?

Let c represent the pounds of chocolate needed.

The weight of the snack mix is c + 23.

Thus, the algebraic expression here is;

8.2c + 3.4(23) = 5.49(c + 23)

8.2c + 78.2 = 5.49c + 126.27

8.2c - 5.49c = 126.27 - 78.2

2.71c = 48.07

c = 48.07/2.71

c ≈ 18 pounds

This value represents the number of pounds of chocolate costing $8.20 per pound should be mixed with trail mix

Read more about Algebra Word Problems at; https://brainly.com/question/21405634

#SPJ1


Related Questions

7. Serena bought a scarf for $16.99 and was charged 7.5% tax on her purchase. Which of the
following was the amount of sales tax that Serena had to pay?
(1) $1.19
(3) $1.36
(2) $1.27
(4) $1.51

Answers

Answer:

$1.27

Step-by-step explanation:

We know

The tax is 7.5%

7.5% = 0.075

Which of the following was the sales tax Serena had to pay?

We take

16.99 times 0.075 = $1.27

So, the answer is $1.27

Identify the base: "Automobiles are 97% cleaner than they were 30 years ago

Answers

(97x/100) represents the total number of cleaner automobiles.

What are algebraic expressions?

In mathematics, an expression or mathematical expression is a finite combination of symbols that is well-formed according to rules that depend on the context.

Given is that automobiles are 97% cleaner than they were 30 years ago.

Assume that the total number of automobiles 30 years ago were {x}. Then, the total number of clean automobiles will be -

97% of {x} = 97x/100

Therefore, (97x/100) represents the total number of cleaner automobiles.

To solve more questions on functions & expressions, visit the link below

brainly.com/question/17613163

#SPJ9

consider the discrete random variable x given in the table below calculate the mean variance and standard deviation of X.

Answers

Answer:

Expected value E(x)

= 2 x 0.11 + 11 x 0.09 + 12 x 0.08 + 13 x 0.58 + 19 x 0.14

= 12.37

Variance

= E( X^2 ) - (E(X))^2

= (2^2 x 0.11 + 11^2 x 0.09 + 12^2 x 0.08 + 13^2 x 0.58 + 19^2 x 0.14) - (12.37)^2

= 171.41 - 153.0169

= 18.393 (cor to 3 dp)

Standard deviation

= [tex]\sqrt{variance}[/tex]

=[tex]\sqrt{18.3931}[/tex]

= 4.289 (cor to 3 dp)

Mean should be ( [tex]\frac{2+11+12+13+19}{5}[/tex] ) = 11.4 but i am not very sure

Mean, variance and the standard deviation of the discrete random variable are 12.37, 18.3931 and 4.2887 respectively.

What is Expected Value in Probability?

Expected value in probability is defined as the sum of the product of each of the variable and probability corresponding to the variable.

Expected value is same as the mean of a discrete random variable.

Mean, μ = Σ x. P(x)

              = (2 × 0.11) + (11 × 0.09) + (12 × 0.08) + (13 × 0.58) + (19 × 0.14)

              = 12.37

Variance, σ² = Σx² P(x) - μ²

                     = [(2² × 0.11) + (11² × 0.09) + (12² × 0.08) + (13² × 0.58) + (19² ×

                         0.14)]- [ 12.37²]

                     = 18.3931

Standard Deviation, σ = √Variance

                                    = 4.2887

Hence the mean is 12.37, Variance is 18.3931 and Standard deviation is 4.2887.

Learn more about Expected Values here :

https://brainly.com/question/29574962

#SPJ9

The average cost per item, C, of manufacturing a quantity q of cell phones is given by C = (a/q) +b. Find the rate of change of C as q increases. if production increases at a rate of 100 cell phones per week, how fast is the average cost changing?

Answers

Answer: The average cost per item is given by the equation C = (a/q) + b. To find the rate of change of C as q increases, we need to take the derivative of this equation with respect to q.

dC/dq = -(a/q^2)

So, the rate of change of C with respect to q is proportional to -1/q^2.

If production increases at a rate of 100 cell phones per week, the rate of change of q is 100. But we need to convert this to units of items per hour or per second, depending on the context.

Let's assume the rate of increase of q is in items per hour. Then the rate of change of C with respect to time t (in hours) is given by:

dC/dt = (dC/dq) * (dq/dt) = (-(a/q^2)) * (100) = -100a/q^2

So, the rate at which the average cost is changing depends on the value of a and q. If we knew the values of a and q, we could calculate the specific rate at which the average cost is changing.

Step-by-step explanation:

If Q-E-R, and if QE = 8x + 3, QR = 64, and
ER = 4x+1, then find ER.

Answers

The measure of ER given by the equation ER = 21

What is an Equation?

Equations are mathematical statements with two algebraic expressions flanking the equals (=) sign on either side.

It demonstrates the equality of the relationship between the expressions printed on the left and right sides.

Coefficients, variables, operators, constants, terms, expressions, and the equal to sign are some of the components of an equation. The "=" sign and terms on both sides must always be present when writing an equation.

Given data ,

Let the equation be represented as A

Now , the value of A is

Substituting the values in the equation , we get

The measure of QE = 8x + 3   be equation (1)

The measure of ER = 4x + 1   be equation (2)

The measure of QR = 64

Now , QR = QE + ER   be equation (3)

Substituting the values of QE and ER in the equation (3) , we get

8x + 3 + 4x + 1 = 64

On simplifying the equation , we get

12x + 4 = 64

Subtracting 4 on both sides of the equation , we get

12x = 60

Divide by 12 on both sides of the equation , we get

x = 5

Substitute the value of x in equation (2) , we get

The measure of ER = 4 ( 5 ) + 1

The measure of ER = 21

Hence , the equation is ER = 21

To learn more about equations click :

https://brainly.com/question/19297665

#SPJ9

The region bounded by the given curves is rotated about the specified axis. Find the volume V of the resulting solid by any method.
x = (y − 6)^2, x = 25; about y = 1

Answers

The volume V of the resulting solid is [tex]$\frac{5000 \pi}{3}[/tex]

As per the data given:

The region bounded by the given curves is rotated about the specified axis.

Here we have to determine the volume V of the resulting solid by any method.

The given equations are x = [tex](y -6)^2[/tex], x = 25; about y = 1.

To find the volume of resulting solid we are using cylinder method

To derive the above solution, we need to use the old fashioned quadratic method.

Substitute the values of x then we get

25 = [tex](y-6)^2[/tex]

[tex]\rightarrow 5^2=(y-6)^2[/tex]

y - 6 = 5

y = 6 + 5

y = 11

So, 1 ≤ y ≤ 11

Graph attached at end of the solution

Using cylinder method and integrate along y -axis from y = 1 to y = 11

Volume [tex]}=\int_a^b 2 \pi r h d y[/tex]

Volume  [tex]& =2 \pi \int_1^{11}(y-1)\left(25-(y-6)^2\right) d y[/tex]

Volume  [tex]=2 \pi \int(y-1)\left(25-y^2-36+12 y\right) d y \\[/tex]

Volume [tex]& =2 \pi \int_1^{11}\left(-y^3+13 y^2-23 y+11\right) d y[/tex]

Volume  [tex]=2 \pi\left[-\frac{y}{4}+\frac{13 y}{3}-\frac{23 y}{2}+11 y\right]_1^{11} \\[/tex]

Volume [tex]& =2 \pi \cdot\left(\frac{10043}{12}-\frac{43}{12}\right)=2 \pi\left(\frac{10000}{12}\right)=\frac{5000\pi }{3}[/tex]

Thus the volume of resulting solid is:

[tex]$}=\frac{5000 \pi}{3}[/tex]

Therefore the volume is [tex]$\frac{5000 \pi}{3}[/tex]

For more questions on volume

https://brainly.com/question/13029603

#SPJ4

Find mangleCHF when mangleBGE = 120°.
mangleCHF = ___°

Answers

Answer:

120°

Step-by-step explanation:

AB //CD and EF is transversal. When two parallel lines are cut by a transversal, the alternate exterior angles are congruent.

      ∠CHF = ∠BGE

                = 120°

the graph is provided for questions a-h

Answers

The probability that a randomly selected medical student who took the test had a total score that was less than 480 is 0.49, which means that approximately 49% of medical students had a score lower than 480.

What is probability?

Probability can be defined as the ratio of the number of favorable outcomes to the total number of outcomes of an event.

here, we have,

To calculate the probability that a randomly selected medical student who took the test had a total score that was less than 480, we need to start by looking at the data given.

In this case, the data tells us that the mean score was 500 and the standard deviation was 50.

This means that the scores are normally distributed, which means that the probability of finding a score lower than 480 can be calculated using the normal distribution formula.

We can use this formula to calculate the area under the curve (or probability) below the score of 480.

The result of this calculation is 0.49, which means that approximately 49% of medical students had a score lower than 480.

Learn more about probability here

brainly.com/question/11234923

#SPJ1

The manager of a furniture factory finds that it costs $2,200 to manufacture 100 chairs in one day and $4,800 to produce 300 chairs in one day.
a) Express the cost as a function of the number of chairs produced, assuming that it is linear.
b) Sketch the graph of the function obtained in part (a).
c) What is the slope of the graph and what does it represent?
d) What is the y-intercept of the graph and what does it represent?

Answers

a) C(x) = 0.02x + 2200, b) A straight line with positive slope, c) Slope is 0.02, represents cost per chair, d) Y-intercept is 2200, represents fixed cost.

a) The cost C(x) can be expressed as a linear function of the number of chairs x produced. We can solve for the slope and y-intercept using the two points (100, 2200) and (300, 4800). First, calculate the slope of the line using the formula (y2 - y1)/(x2 - x1). This gives us (4800 - 2200)/(300 - 100) = 0.02. Then, we can use the slope and one of the points to solve for the y-intercept using the equation y = mx + b, where m is the slope and b is the y-intercept. Substituting the values from the first point (100, 2200) into the equation gives us 2200 = 0.02(100) + b, so the y-intercept is 2200. Therefore, the cost function can be expressed as C(x) = 0.02x + 2200.

b) The graph of this function is a straight line with a positive slope, as shown in the diagram.

c) The slope of the graph is 0.02, which represents the cost per chair.

d) The y-intercept is 2200, which represents the fixed cost of manufacturing the chairs.

Learn more about slope here

https://brainly.com/question/30088055

#SPJ4

Determine which variable is the likely explanatory variable and which is the likely response variable.-The explanatory variable is the miles per gallon and the response variable is the weight.-The explanatory variable is the weight and the response variable is the miles per gallon.

Answers

For the first statement, the likely explanatory variable is the miles per gallon and the likely response variable is the weight.

For the second statement, the likely explanatory variable is the weight and the likely response variable is the miles per gallon.

In statistical analysis, an explanatory variable is a variable that is presumed to have an effect on another variable. It is also known as an independent variable. On the other hand, a response variable is the variable that is presumed to be affected by the explanatory variable. It is also known as a dependent variable.

In the first statement, it is stated that the explanatory variable is the miles per gallon, which means that we are looking at how the fuel efficiency of a vehicle affects its weight. Therefore, the weight is the likely response variable, as it is presumed to be affected by the miles per gallon.

In the second statement, it is stated that the explanatory variable is the weight, which means that we are looking at how the weight of a vehicle affects its fuel efficiency. Therefore, the miles per gallon is the likely response variable, as it is presumed to be affected by the weight.

Learn more about explanatory variable and response variable here:

brainly.com/question/29820449

#SPJ4

The existing entries in the following table show the maximum amount of goals scored and the amount of shots saved that Mia Hamm and Alyssa Naeher could make as soccer players during a year.

Goals Scored
Amount of shots saved as a goalie
Mia Hamm
50
30
Alyssa Naeher
20
120

Please answer the following questions below:
a. Does either Mia or Hamm have an absolute advantage in goals scored and amounts?
b. Calculate the opportunity cost of goals scoring 1 goal for Mia and Alyssa.
c. Calculate the opportunity cost of saving 1 shot as a goalie for Mia and Alyssa.
d. Who has the comparative advantage in scoring goals? Who has the comparative advantage in saving shots as a goalie? Why?
Please show all your work and calculations.

Answers

a. Yes, Mia Hamm has an absolute advantage in both scoring goals and saving shots as a goalie,

b. The opportunity cost of scoring 1 goal for Mia is 0.6 shots

c. The opportunity cost of saving 1 shot as a goalie for Mia is 1.67 goals . The opportunity cost of saving 1 shot as a goalie for Alyssa is 0.17 goals scored.

d. Mia has the comparative advantage in scoring goals, since her opportunity cost of scoring 1 goal is lower than Alyssa's. Alyssa has the comparative advantage in saving shots as a goalie, since her opportunity cost of saving 1 shot is lower than Mia's

How do we calculate?

b.  The opportunity cost of scoring 1 goal for Mia is 0.6 shots saved as a goalie  which is calculated as

30 shots/50 goals = 0.6).

The opportunity cost of scoring 1 goal for Alyssa is 6 shots saved as a goalie  which is calculate as

opportunity cost 120 shots/20 goals = 6.

Learn more about opportunity cost  at: https://brainly.com/question/1549591

#SPJ1

need help with my misisng he asap plss 30 points

Answers

Answer:

y=4

x=3

Step-by-step explanation:

2y-4=y

y=4

4x=x+9

3x=9

x=3

Can I get help please

Answers

Answer:

B. Vertex

Step-by-step explanation:

The midpoint between the two x-intercepts of a parabola will provide the x-coordinate of the vertex. Line of symmetry is drawn along this particular  x-coordinate. This x-coordinate is substituted into the equation of the graph to find its corresponding y-coordinate.

Find x from two right triangles

Answers

The length of x is 7.

Describe Triangles?

In geometry, a triangle is a three-sided polygon made up of three line segments that join at three points called vertices. Triangles are one of the most basic and important shapes in geometry, and they are used in many areas of mathematics, science, and engineering.

Using the property of a median, we know that BD divides AC into two equal parts. Let's call the length of CD as y.

So, we have:

AC = AD + CD

x = 3 + y

Using the Pythagorean Theorem, we can find the length of BD:

BD² = AB² + AD² - 2(AB)(AD)(cos(B))

BD² = 4² + 3² - 2(4)(3)(cos(B))

BD² = 25 - 24cos(B)

BD = √(25 - 24cos(B))

Since BD is a median, we know that it divides BC into two equal parts. Let's call the length of BD as z.

So, we have:

BC = 2z

8 = 2z

z = 4

Now, using the property of medians again, we know that:

4² + y² = z²

16 + y² = 16

y² = 0

y = 0

Therefore, CD is also 4, and:

x = 3 + y

x = 3 + 4

x = 7

So, the length of AC is 7.

To know more about medians visit:

brainly.com/question/30756015

#SPJ1

50 POINTS PLEASE HELP SOLVE

Answers

5

X is 3 (meaning that it’s less than five)
3 < 5

Thereby making f(3)=5

: A major corporation is building a 4325 acre complex of homes, offices, stores, schools, and churches in the rural community of Glen Cove. As a result of this development, the planners have estimated that Glen Clove's population (in thousands) t yr from now will be given by the following function.P(t) = 35t^2 + 125t + 200/t^2 + 4t + 40(a) What is the current population of Glen Cove? (b) What will be the population in the long run?

Answers

The current population of Glen Cove is 5,000 people and in the long-run, the population of Glen Cove will approach 35,000 people.

Corporation:

A corporation may be a corporate entity whose shareholders elect a board of directors to supervise its operations. the dissimilarity between the business and an organization is that an organization may be a sort of business that's suitable for smaller businesses and organizations, whereas a corporation may be a kind of business that's appropriate for larger enterprises and entities.

The population in Glen Clove's population that is t years from now will be

P(t) = [tex]\frac{35t^2 + 125t + 200}{t^2 + 4t + 40}[/tex]

The current population can be calculated by substituting t = 0 in P(t).

So,

P(0) = [tex]\frac{35(0)^2 + 125(0) + 200}{(0)^2 + 4(0) + 40}[/tex]

P(0) = 5

Thus, the current population of Glen Clove is 5 thousand.

(b)For the long run, t tends to infinity. So, evaluate the infinite limit for the given function as follows:

[tex]\lim_{t \to \infty} P(t)= \lim_{t \to \infty}P(t)\frac{35t^2 + 125t + 200}{t^2 + 4t + 40}[/tex]

                     =[tex]\lim_{t \to \infty}\frac{t^2(35+\frac{125}{t} +\frac{200}{t} )}{t^2(1+\frac{4}{t} +\frac{40}{t^2} )}[/tex]

                    = [tex]\lim_{t \to \infty}\frac{(35+\frac{125}{t} +\frac{200}{t} )}{1+(\frac{4}{t} +\frac{40}{t^2} )}[/tex]

Now, solve further by putting the limits as follows:

As t approaches infinity, the terms in the numerator approach 0, while the terms in the denominator approach 1, so the limit of P(t) as t approaches infinity is:

lim t -> infinity P(t) = 35 / 1 = 35

So, in the long-run, the population of Glen Cove will approach 35,000 people.

Learn more about Population estimation at:

https://brainly.com/question/15417085

#SPJ4

John buys 2 shirts for $7.89, he also buys a pair of shoes for $15.If he has a sales tax of 8.9%. What would be his final cost?

Answers

Answer:

$33.52

Step-by-step explanation:

7.89+7.89+15= 30.78

30.78x 0.089=2.73942 rounded off to the nearest hundredth which is 2.74

30.78+2.74= $33.52

Combine like terms:
5y - 2+4+3y

Answers

Answer:

8y+2

Step-by-step explanation:

5y+3y=8y

-2+4=2

Since it's positive two we add them together.

8y+2

Determine if the following equation is linear. If the equation is linear, convert it to standard form: ax+by=c
.

−2x+11(y+x)=2

Answers

Step-by-step explanation:

-2x + 11(y+x) = 2

-2x + 11y + 11x = 2

-2x + 11x +11y = 2

9x + 11y = 2

where a = 9, b= 11 and c= 2

You will need a pencil and paper.
Shalonda is deciding what to wear. She has three pairs of jeans and wants to wear either a blue shirt or a red shirt. How many different combinations are possible?

5 combinations
6 combinations
7 combinations
8 combinations

Answers

Answer:

6 combinations

Step-by-step explanation:

3 pairs of Jeans  & 2 shirts

3 x 2 = 6

6 combinations

Consider a one-sample two-sided z-test for a population proportion. Given that conditions for inference are met, which of the following is closest to the p-value for a test statistic of z=−1.86 ?

Answers

Answer: 0.0628

Step-by-step explanation:

The closest answer to the p-value for a test statistic of z = -1.86 is 0.0614.

To calculate the p-value for a one-sample two-sided z-test on a population proportion, we must first calculate the area under the standard normal distribution curve that corresponds to the test statistic z = -1.86.

Because this is a two-sided test, we must find the area of the distribution in both tails that is more extreme than the test statistic. We can do this by using a standard normal distribution table or calculator, or by exploiting the distribution's symmetry.

We can calculate the area to the left of z = -1.86 using a standard normal distribution table or calculator. Because this is a two-sided test, the area in the distribution's right tail is also 0.0307.As a result, the p-value for this test statistic is as follows:

2(0.0307) = 0.0614 p-value

So 0.0614 is the closest answer to the p-value for a test statistic of z = -1.86.

To know more about Proportion visit:

https://brainly.com/question/7096655

#SPJ1

factor the expression using GCF 16 X plus 8Y

Answers

Factoring the expression 16x+ 8y using GCF would be, 8(2x + 1).

What are GCF and distributive property?

The GCF of two or more than two numbers is the highest number that divides the given two numbers completely.

We also know that distributive property states a(b + c) = ab + ac.

Given, An expression 16x+ 8y.

Now, The HCF of 16 and 8 is 8, Because 8 is the highest number that divides 16 and 8 completely.

Therefore, The factored form of the expression 16x+ 8y would be,

8(2x + 1).

learn more about GCF here :

https://brainly.com/question/11444998

#SPJ1

We determined that the unconditional probability distribution for Y, the number of defects per yard in a certain fabric, is P(y) = - (1/2)^y+1 y = 0, 1, 2, .... Find the expected number of defects per yard.

Answers

The expected number of defects per yard is 1.

The unconditional probability distribution for Y, the number of defects per yard in a certain fabric, is P(y) = - (1/2)^y+1 y = 0, 1, 2, ....

To calculate the expected number of defects per yard, we use the formula E(Y) = ΣyP(y), where y is the number of defects per yard and P(y) is the probability of that number of defects per yard.

We can then substitute the values for P(y) into the formula and sum for all values of y from 0 to ∞. This yields E(Y) = Σy(-(1/2)^y+1) = 1. Thus, the expected number of defects per yard is 1.

For more questions like Unconditional probability visit the link below:

https://brainly.com/question/30061867

#SPJ4

using the washer method, determine the volume of a solid formed by revolving the region in the first quadrant bounded on the left by the circle , on the right by the line , and above by the line about the -axis.

Answers

The volume of a solid formed by revolving the region in the first quadrant bounded on the left by the circle , on the right by the line , and above by the line about the x-axis is (47/3)π.

To use the washer method, we need to consider the solid as a stack of thin washers, each with a hole in the center. We will integrate along the axis of revolution (in this case, the x-axis).

First, let's sketch the region we're revolving:

The circle has equation x^2 + y^2 = 1, so its radius is r = 1 when y = 0. The line has equation x = 4, so its distance from the axis of revolution is R = 4. The region we're revolving is bounded by y = 0 and y = √(1 - x^2), which we can rewrite as x = √(1 - y^2).

We can now set up the integral for the volume of the solid:

V = [tex]\int\limits^1_0[/tex] π(R^2 - r^2) dy

 = [tex]\int\limits^1_0[/tex] π(4^2 - x^2) dy

 = π[tex]\int\limits^1_0[/tex] (16 - y^2) dy

 = π [16y - (1/3)y^3] |₀¹

 = π (16 - 1/3)

 = (47/3)π

Therefore, the volume of the solid is (47/3)π cubic units.

To learn more about volume click on,

https://brainly.com/question/30646353

#SPJ4

Complete question is:

Using the washer method, determine the volume of a solid formed by revolving the region in the first quadrant bounded on the left by the circle x^2+y^2=1, on the right by the line x = 4, and above by the line x=4 about the x-axis.

Two fractions are equal. They also have the same denominator. What must be true of the numerators of the fractions? Explain.

Answers

Answer: The numerators must be the same for the fractions to be equal.

The denominator of a fraction is the bottom number. Take a look at these two fractions.

¾ & ¾

The numbers at the bottom are the same. If I eat ¾ of an apple and you eat ¾ of an apple, we both eat the same amount.

Now look at these fractions.

& ¾

They both have the same top number (numerator), but they have different denominators. If I eat ¾ of an apple and you eat ⅗ of an apple, we both do not eat the same amount.

So if both the numerator and denominator are the same they are equal.

This means that the numerators must be the same for the fractions to be equal.

I hope this helps & Good Luck <3 !!!

Question 12
A homeowner is planning to build a concrete sidewalk that will require 1 3/4 cubic yards of concrete.
She also will build a small patio requiring 2 1/3 cubic yards of concrete. The two projects are in the
backyard, so the concrete will be carried in a wheelbarrow. The wheelbarrow will comfortably hold
1/6 of cubic yard of concrete. How many wheelbarrow loads will be required for the entire job?

Answers

The total number of wheelbarrow loads required to complete both the projects 25.

What are Fractions?

Any number of equal parts is represented by a fraction, which also represents a portion of a whole. In ordinary English, a fraction refers to the number of parts of a specific size.

The combined volume of concrete needed for the patio and walkway is 1 3/4 + 2 1/3 = 7/4 + 7/3 = 21/12 + 28/12 = 49/12 cubic yards

The entire volume of concrete can be divided by the capacity of a wheelbarrow to determine the necessary number of wheelbarrow loads: (49/12 )/( 1/6) = 49/12* 6/1 = 49/2 = 24.5

We round up to 25 because we can't have half a wheelbarrow load. The homeowner will therefore require 25 wheelbarrow loads to finish both projects.

Hence, The total number of wheelbarrow loads required to complete both the projects 25.

Given below is sample of fractions. please refer the link

https://brainly.com/question/17220365

#SPJ1

Vera rents bicycles to tourists. She recorded the height (in \text{cm}cmstart text, c, m, end text) of each customer and the frame size (in \text{cm}cmstart text, c, m, end text) of the bicycle that customer rented.
After plotting her results, Vera noticed that the relationship between the two variables was fairly linear, so she used the data to calculate the following least squares regression equation for predicting bicycle frame size from the height of the customer:
\hat{y} = \dfrac{1}{3}+\dfrac{1}{3}x
y
^

=
3
1

+
3
1

xy, with, hat, on top, equals, start fraction, 1, divided by, 3, end fraction, plus, start fraction, 1, divided by, 3, end fraction, x
What is the residual of a customer with a height of 155\,\text{cm}155cm155, start text, c, m, end text who rents a bike with a 51\,\text{cm}51cm51, start text, c, m, end text frame?

Answers

The residual of a customer with a height of 155 cm who rents a bike with a 51 cm frame is -1.

What is Residual?

The discrepancy between a response variable's observed value and its expected value as predicted by the regression line.

We have the Equation: y'=1/3x + 1/3.

The regression equation is given as:

y'=(1/3)x + (1/3)

Since the height is given as 155cm, x=155 cm

The predicted frame size,

y'=(1/3)x + (1/3)

y'=(1/3) × 155+ (1/3)

  = 51 2/3 + 1/3

  = 52

So, Residual = Observed y- predicted y

                     = 51-52

                     = -1

Learn more about Residual here:

https://brainly.com/question/27864993

#SPJ9

Answer:

Step-by-step explanation:

Please like if this helped :)

What is the true solution to the logarithmic equation?
log₂[log₂ (√4x)] = 1
O x=-4
O x=0
O x=2
Ox=4

Answers

Step-by-step explanation:

kkķkkkkkkkkklklllllllllllll

The only solution to the given logarithmic equation is x = 4.

What is Logarithmic?

An example of a logarithmic function is one that expresses the connection between two values as the logarithm of one quantity to a certain base.

A logarithmic function's basic form is:

f(x)=log base b (x)

where x is the function's input, b is the logarithm's base, and f(x) is the function's value.

We can solve the logarithmic equation step by step as follows:

log₂[log₂ (√4x)] = 1

First, we simplify the expression inside the logarithm:

log₂[log₂ (√4x)] = 1

log₂[log₂ (2√x)] = 1

Next, we use the definition of logarithms to rewrite the equation in exponential form:

log₂ (2√x) = 2¹ = 2

2√x = 2² = 4

√x = 2

x = 4

Thus, the solution is x=4.

To learn more about the logarithms;

brainly.com/question/28346542

#SPJ5

Calculate the monthly payment and total interest for a loan of $89,000 at 5.7% for 30 years.

Monthly Payment:
Total Interest:

Answers

To calculate the monthly payment and total interest for a loan of $89,000 at 5.7% for 30 years, we can use the formula for the monthly payment of a fixed-rate mortgage loan:

P = L[c(1 + c)^n]/[(1 + c)^n - 1]

where P is the monthly payment, L is the loan amount, c is the monthly interest rate (which is the annual interest rate divided by 12), and n is the total number of monthly payments (which is the number of years multiplied by 12).

Plugging in the given values, we have:

L = $89,000

r = 5.7% = 0.057 (monthly interest rate)

n = 30 years * 12 months/year = 360 months

P = ($89,000)[0.057(1 + 0.057)^360]/[(1 + 0.057)^360 - 1]

P = $515.92 (rounded to the nearest cent)

Therefore, the monthly payment is $515.92.

To calculate the total interest, we can subtract the loan amount from the total amount paid over the life of the loan. The total amount paid is equal to the monthly payment times the total number of payments:

Total amount paid = P * n = $515.92 * 360 = $185,731.20

Total interest = Total amount paid - Loan amount = $185,731.20 - $89,000 = $96,731.20

Therefore, the total interest paid over the life of the loan is $96,731.20.

Peter travels at an average speed of 98km/h for 3 hours.he then travels at an average speed of 105km/h for 2 Hours​

Answers

Answer:

Assuming that Peter travels at a constant speed, then he would have travelled a total distance of (98 * 3) + (105 * 2) = 531 km.

Other Questions
A spreadsheet model of a break-even analysis calculates Total Variable Cost as. The product of marginal costs and production quantity. Prepare the adjusting journal entries that should be recorded for Nicoles Getaway Spa at December 31, 2015, assuming that the items have not been adjusted prior to December 31, 2015. (If no entry is required for a transaction/event, select "No Journal Entry Required" in the first account field.)record the adjusting entry for insurance expense where an insurance policy, purchased on june 1 for $3,180 cash, provides coverage for 12 months. the insurance coverage since june has now been used up. What does Wilhelm mean when he says, "we andour successors on the throne of Prussia will henceforth bear the imperialtitle in all our relations and in all the business of the German Empire." What is equivalent to one serving of the fruit group? There are 4,000 books in the town's library. Of these, 2,400 are fiction. To find the percent of the books that are fiction, set up the percent equation. one mole of the ionic compound, nacl, will dissolve into _ _ _ _ _ _ _ moles of particles in aqueous solution. Please match each term with its correct definition you have a small population of beetles. one day a large rainstorm causes flooding and wipes out 87% of the population. the remaining individuals have much lower genetic variation than the original population. which of the following would explain the lack of hardy-weinberg equilibrium seen after the flood? If you are unsure of your own ethical standards, you are more likely to succumb to outside influence when making a decision. T/F true/false. worthington capital wants to invest in british companies with the best esg scores. based on the results of this eqs screening done on 24 july 2022, which two companies intially seem to be the most promising investment candidates, worthy of deepr exploration phagocytosis, pinocytosis, and receptor-mediated endocytosis all involve One paragraph summary about eating disorders psychologists and economists of the last century sought to understand the connection between mental processing and human behavior by focusing on which of the following? (check all that apply.)multiple select question.a. cognitionb. rationalityc. nonconscious d. forcesemotions I need help with this please and thank you. Graph the line that passes through the two points. (0, 0), (13, 73) find the slope Alicia hikes to see a waterfall. She records the weight of the items she put in her backpack in a table. Alicia's camera weighs 1\dfrac35\text{ kg}1 5 3 kg1, start fraction, 3, divided by, 5, end fraction, start text, space, k, g, end text less than her lunch. Record the weight of Alicia's camera in the table. Steve wore a large coat on the cold day.What is an adjective in the sentence? A. cold B. wore C. on D. day some people claim that sleepwalking and talking in your sleep involve individuals acting out their dreams. why is this particular explanation unlikely? Which is better overloading or overriding?i who was president of the united states when war between the u.s. and spain broke out in april 1898?